![]() |
|
|||
|
|
|
|
|
|
|
USMLE Step 2 CK Forum USMLE Step 2 CK Discussion Forum: Let's talk about anything related to USMLE Step 2 CK exam |
![]() |
|
Thread Tools | Search this Thread | Display Modes |
#1
|
|||
|
|||
![]()
Q- Which one of the following treatment most likely prevents meningeal leukemia in a young boy diagnosed with acute lymphoblastic leukemia?
A- Cyclophosphamide B- High dose of cortisone C- Intrathecal methotrexate D- Radiotherapy Q- An old man presents with acute severe back pain, tachycardia, hypotension and syncope. Which of the following is the most likely diagnosis? A- Acute mesenteric ischemia B- Acute myocardial infarction C- Pulmonary embolism D- Rupture of abdominal aortic aneurysm Q- A group of people are assigned for a research study about the effect of a new hypoglycemia medication. Because they are aware of the study, they eat less carbohydrate. Which type of bias is involved in this study? A- Confounding bias B- Hawthorne effect C- Information bias D- Sample distortion bias Answer and Explanation |
|